Solar Power

This topic has expert replies
Senior | Next Rank: 100 Posts
Posts: 42
Joined: Tue Feb 16, 2016 2:50 am
Thanked: 1 times
Followed by:1 members

Solar Power

by dustystormy » Sat Sep 03, 2016 2:00 pm
United States manufacturers currently produce most of the world's solar-power generators--most of which are exported to Europe. However, European manufacturers are emerging and probably will ultimately capture much of the European market. The United States government is funding initiatives intended to encourage use of solar power within the United States. If these initiatives succeed in increasing the demand for solar-power generators in the United States, United States manufacturers will probably maintain significant production levels, since __________.

Which of the following most logically completes the argument?

(A) some United States manufacturers have been substantially increasing their output over the last several years

(B) the efficiency of solar-power generators in converting energy from the Sun into electric power is not improving as fast as it once did

(C) just as European manufacturers enjoy certain competitive advantages in Europe, so do United States manufacturers in the United States

(D) European governments are currently undertaking initiatives to stimulate the use of solar power within Europe

(E) the current market for solar-power generators in the United States is very limited

[spoiler]OA:C
Experts can you please explain why [E] is wrong?[/spoiler]

User avatar
Senior | Next Rank: 100 Posts
Posts: 53
Joined: Wed Jun 04, 2014 10:50 am
Thanked: 6 times
Followed by:2 members

by prabsahi » Sat Sep 03, 2016 7:08 pm
Option E.
The current market for solar power generators in the US is very limited.
We cannot comprehend from this statement --what is the current scenario and what could be the potential after
the inititatives taken by government.

For example assume our current production of US generators is 500 million.

What if US has a potential of 100 million generators and currently its market is of only 20 million.The rest can be supplied by our current production level.(our capacity is 500).But our production will still have a surplus.
So the extent of current market limited will not help us.Hence option E is wrong.Hope it helps!!

User avatar
GMAT Instructor
Posts: 15539
Joined: Tue May 25, 2010 12:04 pm
Location: New York, NY
Thanked: 13060 times
Followed by:1906 members
GMAT Score:790

by GMATGuruNY » Sat Sep 03, 2016 7:09 pm
dustystormy wrote:United States manufacturers currently produce most of the world's solar-power generators--most of which are exported to Europe. However, European manufacturers are emerging and probably will ultimately capture much of the European market. The United States government is funding initiatives intended to encourage use of solar power within the United States. If these initiatives succeed in increasing the demand for solar-power generators in the United States, United States manufacturers will probably maintain significant production levels, since __________.

Which of the following most logically completes the argument?

(A) some United States manufacturers have been substantially increasing their output over the last several years

(B) the efficiency of solar-power generators in converting energy from the Sun into electric power is not improving as fast as it once did

(C) just as European manufacturers enjoy certain competitive advantages in Europe, so do United States manufacturers in the United States

(D) European governments are currently undertaking initiatives to stimulate the use of solar power within Europe

(E) the current market for solar-power generators in the United States is very limited
The correct answer choice must support the conclusion that United States manufacturers will probably maintain significant production levels of solar generators even though European manufacturers are emerging and probably will ultimately capture much of the European market.

C: Just as European manufacturers enjoy certain competitive advantages in Europe, so do United States manufacturers in the United States.
Here, United States manufacturers enjoy competitive advantages in the United States, SUPPORTING the conclusion that United States manufacturers will probably maintain significant production levels, even though European manufacturers are emerging and probably will ultimately capture much of the European market.

The correct answer is C.

E: The current market for solar-power generators in the United States is very limited.
The current market is irrelevant.
The conclusion is constrained to what will happen if...initiatives succeed in increasing the demand for solar-power generators in the United States.
Eliminate E.
Private tutor exclusively for the GMAT and GRE, with over 20 years of experience.
Followed here and elsewhere by over 1900 test-takers.
I have worked with students based in the US, Australia, Taiwan, China, Tajikistan, Kuwait, Saudi Arabia -- a long list of countries.
My students have been admitted to HBS, CBS, Tuck, Yale, Stern, Fuqua -- a long list of top programs.

As a tutor, I don't simply teach you how I would approach problems.
I unlock the best way for YOU to solve problems.

For more information, please email me (Mitch Hunt) at [email protected].
Student Review #1
Student Review #2
Student Review #3

User avatar
Legendary Member
Posts: 2131
Joined: Mon Feb 03, 2014 9:26 am
Location: https://martymurraycoaching.com/
Thanked: 955 times
Followed by:140 members
GMAT Score:800

by MartyMurray » Sat Sep 03, 2016 7:30 pm
dustystormy wrote:[spoiler]Experts can you please explain why [E] is wrong?[/spoiler]
First, clearly understand what the prompt is saying.

- U.S. manufacturers produce most solar power generators.

- Most of those generators are exported to Europe.

- European manufactures will ultimately capture much of the European market.

OK. So far, we have reason to believe that U.S. manufacturers of solar power equipment will sell fewer units as they lose business in Europe.

- The U.S. government is encouraging use of solar power in the U.S.

Conclusion: If the government initiatives succeed in increasing demand for solar-power generators, U.S. manufacturers will maintain significant production levels.

Premise: Because ___________.

(E) the current market for solar-power generators in the United States is very limited

This is a trap answer. Why? Because you can construe it as indicating that there is a lot of room for growth of the market for solar power generators in the U.S., and therefore come to the conclusion that U.S. manufacturers will see sales growth in the U.S.

However, while it does indicate that there is room for the U.S. market to grow, the prompt says that European manufacturers are emerging. It's possible that the European manufacturers will outcompete the U.S. manufacturers in the U.S. as well.

Meanwhile, the argument in the prompt is QUALIFIED, saying "If these initiatives succeed in increasing demand ..." So to support the conclusion, we don't need something that indicates that demand will increase. Rather we need something that shows that IF demand does increase U.S. manufacturers will maintain significant production levels.

So while E indicates there is room for growth in demand for solar generators in the U.S., it does not provide additional support for the argument that IF the government initiatives succeed in increasing demand, U.S. manufacturers will maintain significant production.

C, on the other hand, provides clear, logical support for the conclusion. Even though European manufactures are expected to dominate in Europe, C provides reason to believe that U.S. manufacturers will dominate in the U.S. and therefore will see sales growth in the U.S. IF the U.S. government initiatives succeed in increasing demand for solar-power generators.
Marty Murray
Perfect Scoring Tutor With Over a Decade of Experience
MartyMurrayCoaching.com
Contact me at [email protected] for a free consultation.

Junior | Next Rank: 30 Posts
Posts: 28
Joined: Wed Jan 06, 2016 6:18 pm

by Alchemist14 » Thu Oct 13, 2016 8:07 pm
Marty and Mitch, Please help me on this one.

If I lose weight, I can run faster
E says something like, I have spoken to fitness experts and it is feasible to lose weight.
C says something like, losing weight increases lung capacity-better inflow and outflow of oxygen, helping you to run faster
Now E gives more info about 'if I lose weight' part so it no way strengthens the argument. C makes the argument more likely.

I remember another question on the same lines.- Here is the conclusion
Therefore, provided this technology is effective, those fears are groundless.

Correct Answer-Other than the possibility of a leak, there is no realistic pollution threat posed to the lake by the pipeline's construction.

Trap Answer-There is no reason to believe that the leak-preventing technology would be ineffective when installed in the pipeline in Lake Konfa.- This choice still talks about 'provided this technology is effective' part.

These two questions use similar style, right? I am not making rules but trying to apply knowledge learnt from one question to another.

Could you guys please let me know whether I am thinking correctly?

Thanks in advance,
Al

User avatar
Legendary Member
Posts: 2131
Joined: Mon Feb 03, 2014 9:26 am
Location: https://martymurraycoaching.com/
Thanked: 955 times
Followed by:140 members
GMAT Score:800

by MartyMurray » Fri Oct 14, 2016 5:03 am
Alchemist14 wrote:Marty and Mitch, Please help me on this one.

If I lose weight, I can run faster
E says something like, I have spoken to fitness experts and it is feasible to lose weight.
C says something like, losing weight increases lung capacity-better inflow and outflow of oxygen, helping you to run faster
Now E gives more info about 'if I lose weight' part so it no way strengthens the argument. C makes the argument more likely.

I remember another question on the same lines.- Here is the conclusion
Therefore, provided this technology is effective, those fears are groundless.

Correct Answer-Other than the possibility of a leak, there is no realistic pollution threat posed to the lake by the pipeline's construction.

Trap Answer-There is no reason to believe that the leak-preventing technology would be ineffective when installed in the pipeline in Lake Konfa.- This choice still talks about 'provided this technology is effective' part.

These two questions use similar style, right? I am not making rules but trying to apply knowledge learnt from one question to another.

Could you guys please let me know whether I am thinking correctly?

Thanks in advance,
Al
All the E answers are wrong for basically the same reason. Yes, you nailed it.

For what it's worth, this, "C says something like, losing weight increases lung capacity-better inflow and outflow of oxygen, helping you to run faster" does not quite sound like a GMAT right answer, in that it does not exactly logically support the idea that a particular person will be able to run faster after losing weight.

I believe a GMAT right answer would be more like "The results of a multi factorial treadmill test (whatever that is) show that only the effects of excess weight on my knees are keeping me from running faster."

That statement provides logical support for the conclusion, support that tends to rule out that there could be reasons other than weight that are preventing you from running faster.
Marty Murray
Perfect Scoring Tutor With Over a Decade of Experience
MartyMurrayCoaching.com
Contact me at [email protected] for a free consultation.

Junior | Next Rank: 30 Posts
Posts: 28
Joined: Wed Jan 06, 2016 6:18 pm

by Alchemist14 » Fri Oct 14, 2016 8:05 am
Marty Murray wrote: All the E answers are wrong for basically the same reason. Yes, you nailed it.

For what it's worth, this, "C says something like, losing weight increases lung capacity-better inflow and outflow of oxygen, helping you to run faster" does not quite sound like a GMAT right answer, in that it does not exactly logically support the idea that a particular person will be able to run faster after losing weight.

I believe a GMAT right answer would be more like "The results of a multi factorial treadmill test (whatever that is) show that only the effects of excess weight on my knees are keeping me from running faster."

That statement provides logical support for the conclusion, support that tends to rule out that there could be reasons other than weight that are preventing you from running faster.
Thanks for the superfast reply. Yes Im really bad at making my own analogies. Have to think a lot. Sometimes it takes 10 mins or more. Are there other ways to improve CR skills other than stating the goal( simplifying the argument ) and making analogies? I dont want to learn quick fixes but want ways to improve myself in terms of thinking as a manager does.

Thanks once again.

Al

User avatar
Legendary Member
Posts: 2131
Joined: Mon Feb 03, 2014 9:26 am
Location: https://martymurraycoaching.com/
Thanked: 955 times
Followed by:140 members
GMAT Score:800

by MartyMurray » Sat Oct 15, 2016 6:12 pm
Alchemist14 wrote:Yes Im really bad at making my own analogies. Have to think a lot. Sometimes it takes 10 mins or more. Are there other ways to improve CR skills other than stating the goal( simplifying the argument ) and making analogies? I dont want to learn quick fixes but want ways to improve myself in terms of thinking as a manager does.

Thanks once again.

Al
Well, you did make a great analogy for E.

Getting right answers to CR questions requires vision. To improve your vision, you could do the following.

For all Weaken, Strengthen and Assumption questions, look for flaws and gaps in the arguments. I am not suggesting that you always do this when you are answering questions while taking the test. I am saying to do this as an exercise for improving your vision. Generally the arguments in those types of questions contain one or more flaws or gaps, and by learning to see the flaws you will train yourself to see more clearly.

Another thing you can do is to define why every answer is either wrong or right. It's one thing to say something along the lines of "I don't see how this answer choice is related to the argument," and another to say "Here is exactly why this answer choice is not the correct one."

I also have people explain how certain wrong answers, trap answers, could seem correct. By learning to see the logic underlying why someone would pick a wrong answer, you can develop your vision and analytical skills and become something along the lines of a GMAT CR insider.
Marty Murray
Perfect Scoring Tutor With Over a Decade of Experience
MartyMurrayCoaching.com
Contact me at [email protected] for a free consultation.

Legendary Member
Posts: 944
Joined: Wed May 30, 2012 8:21 am
Thanked: 8 times
Followed by:5 members

by RBBmba@2014 » Mon Nov 07, 2016 11:40 am
Hi Mitch/Marty/Dave/Ceilidh,

Could you please let me know whether I'm correct as follows:

Option E is wrong because it doesn't provide any NEW information - it ALREADY appears from the ARGUMENT (at least, within the scope of the ARGUMENT) that the current market for solar-power generators in the United States is very limited BECAUSE the ARGUMENT says that most of the US-manufactured solar-power generators are exported to Europe and United States government is funding initiatives intended to encourage use of solar power within the United States .

And a STRENGTHENER should be a NEW information that will make the CONCLUSION more believable.

Correct me please if wrong!

Legendary Member
Posts: 944
Joined: Wed May 30, 2012 8:21 am
Thanked: 8 times
Followed by:5 members

by RBBmba@2014 » Mon Dec 26, 2016 9:27 pm
@ Verbal Experts,
Any thoughts on my above concerns ?

Look forward to hear from you! Much thanks in advance.

User avatar
Legendary Member
Posts: 2663
Joined: Wed Jan 14, 2015 8:25 am
Location: Boston, MA
Thanked: 1153 times
Followed by:128 members
GMAT Score:770

by DavidG@VeritasPrep » Sat Dec 31, 2016 11:50 am
RBBmba@2014 wrote:Hi Mitch/Marty/Dave/Ceilidh,

Could you please let me know whether I'm correct as follows:

Option E is wrong because it doesn't provide any NEW information - it ALREADY appears from the ARGUMENT (at least, within the scope of the ARGUMENT) that the current market for solar-power generators in the United States is very limited BECAUSE the ARGUMENT says that most of the US-manufactured solar-power generators are exported to Europe and United States government is funding initiatives intended to encourage use of solar power within the United States .

And a STRENGTHENER should be a NEW information that will make the CONCLUSION more believable.

Correct me please if wrong!
Well, you're right that a strengthener should provide something that hasn't already been mentioned in the prompt, but in this case, we weren't really told what the current U.S. market for solar polar generators is. (Even if most of what is manufactured domestically is shipped overseas. It could be true that there's a reasonably healthy domestic market, but for whatever reason, the European market is substantially larger.) The real problem with E is that the size of the current U.S. market isn't helpful information. The question is whether 1) the government initiative to boost domestic demand will be adequate to offset whatever stands to be lost in the European market, once European manufacturers capture more market share and 2) will U.S. manufacturers lose new domestic business to European manufacturers?
Veritas Prep | GMAT Instructor

Veritas Prep Reviews
Save $100 off any live Veritas Prep GMAT Course

Legendary Member
Posts: 944
Joined: Wed May 30, 2012 8:21 am
Thanked: 8 times
Followed by:5 members

by RBBmba@2014 » Tue Jan 03, 2017 9:34 pm
DavidG@VeritasPrep wrote: Well, you're right that a strengthener should provide something that hasn't already been mentioned in the prompt, but in this case, we weren't really told what the current U.S. market for solar polar generators is. (Even if most of what is manufactured domestically is shipped overseas. It could be true that there's a reasonably healthy domestic market, but for whatever reason, the European market is substantially larger.)
Hmm..So, let me just confirm whether I got you completely correct -

You're essentially saying that as the ARGUMENT doesn't mention anything EXPLICIT about current market, it could well be true that US got a healthy domestic market and its production level is pretty high that it exports most of the generators even after meeting the demand from the domestic market.

Did I get you properly ?
RBBmba@2014 wrote:Option E is wrong because it doesn't provide any NEW information - it ALREADY appears from the ARGUMENT (at least, within the scope of the ARGUMENT) that the current market for solar-power generators in the United States is very limited BECAUSE the ARGUMENT says that most of the US-manufactured solar-power generators are exported to Europe and United States government is funding initiatives intended to encourage use of solar power within the United States .

And a STRENGTHENER should be a NEW information that will make the CONCLUSION more believable.
However, can we really discard this right away ? It might also be TRUE...I mean, when a country exports MOST of its products (say,any specific PRODUCT CATEGORY) then, I guess, by common sense we can say that TWO possible scenarios could be -

1. The one you've mentioned

2. The one I've mentioned - Country doesn't really have strong DOMESTIC demand,so it exports most of its PRODUCT. (Perhaps, they manufacture that PRODUCT in such a large scale with EXPORT in mind - isn't this logical ?)

DavidG@VeritasPrep wrote:The real problem with E is that the size of the current U.S. market isn't helpful information. The question is whether 1) the government initiative to boost domestic demand will be adequate to offset whatever stands to be lost in the European market, once European manufacturers capture more market share and 2) will U.S. manufacturers lose new domestic business to European manufacturers?
Yes.
Btw, can we say that this is another point as Pre-thinking -- whether U.S. manufacturers will have the capacity/infrastructure to maintain significant production levels to meet the increased demand in the domestic market ? (I sense, it's MAY NOT be correct in this scenario, but just want to get this confirmed!)

User avatar
Legendary Member
Posts: 2663
Joined: Wed Jan 14, 2015 8:25 am
Location: Boston, MA
Thanked: 1153 times
Followed by:128 members
GMAT Score:770

by DavidG@VeritasPrep » Wed Jan 04, 2017 9:29 am
Hmm..So, let me just confirm whether I got you completely correct -

You're essentially saying that as the ARGUMENT doesn't mention anything EXPLICIT about current market, it could well be true that US got a healthy domestic market and its production level is pretty high that it exports most of the generators even after meeting the demand from the domestic market.

Did I get you properly ?
You sure did.
Veritas Prep | GMAT Instructor

Veritas Prep Reviews
Save $100 off any live Veritas Prep GMAT Course

User avatar
Legendary Member
Posts: 2663
Joined: Wed Jan 14, 2015 8:25 am
Location: Boston, MA
Thanked: 1153 times
Followed by:128 members
GMAT Score:770

by DavidG@VeritasPrep » Wed Jan 04, 2017 9:34 am
2. The one I've mentioned - Country doesn't really have strong DOMESTIC demand,so it exports most of its PRODUCT. (Perhaps, they manufacture that PRODUCT in such a large scale with EXPORT in mind - isn't this logical ?)
Perfectly logical. That's not why it's wrong. It's wrong because even if it were true, it wouldn't cause domestic production to remain strong. (But if your point is that you shouldn't dismiss the answer out of hand before considering what impact limited domestic demand would have on domestic production, then we're on the same page.)
Veritas Prep | GMAT Instructor

Veritas Prep Reviews
Save $100 off any live Veritas Prep GMAT Course

User avatar
Legendary Member
Posts: 2663
Joined: Wed Jan 14, 2015 8:25 am
Location: Boston, MA
Thanked: 1153 times
Followed by:128 members
GMAT Score:770

by DavidG@VeritasPrep » Wed Jan 04, 2017 9:39 am
Btw, can we say that this is another point as Pre-thinking -- whether U.S. manufacturers will have the capacity/infrastructure to maintain significant production levels to meet the increased demand in the domestic market ? (I sense, it's MAY NOT be correct in this scenario, but just want to get this confirmed!)
Sure, something that's worth considering. But because domestic capacity seems adequate to meet foreign demand, the bigger issue is whether an increase in domestic demand will offset the decrease in exports. (Remember, the argument is that production will remain robust. In order to remain at its current level, presumably it wouldn't need increased capacity. If a manufacturer produces 100 widgets for country X, it should be able to produce 100 widgets for country Y, should X shift to another supplier.)
Veritas Prep | GMAT Instructor

Veritas Prep Reviews
Save $100 off any live Veritas Prep GMAT Course